3
$\begingroup$

Let $wt(n)$ be A000120, number of $1$'s in binary expansion of $n$ (or the binary weight of $n$) and $$n=2^{t_1}(1+2^{t_2+1}(1+\dots(1+2^{t_{wt(n)}+1}))\dots)$$ Then we have an integer sequence given by $$a(n)=\sum\limits_{j=0}^{2^{wt(n)}-1}m^{wt(n)-wt(j)}\prod\limits_{k=0}^{wt(n)-1}(1+wt(\left\lfloor\frac{j}{2^k}\right\rfloor))^{t_{k+1}+1}, a(0)=1$$ Let $$s(n,m)=\sum\limits_{k=0}^{2^n-1}a(k)$$ then I conjecture that for any $m$ $$s(n,m)=\sum\limits_{k=0}^{n+1}k!{n+1\brace k}(m+1)^{n-k+1}$$ Is there a way to prove it?

Similar questions:

$\endgroup$
5
  • 1
    $\begingroup$ It's worth to link the previous relevant questions. E.g., mathoverflow.net/q/406436 where the formula for $a(n)$ is proved. $\endgroup$ Oct 21, 2021 at 15:37
  • $\begingroup$ @MaxAlekseyev, thank you for comment! Is the link added automatically? $\endgroup$ Oct 21, 2021 at 15:43
  • 1
    $\begingroup$ What do you mean under "automatically"? $\endgroup$ Oct 21, 2021 at 20:15
  • $\begingroup$ @MaxAlekseyev, I tried to delete the link, but I didn't find where to do it. $\endgroup$ Oct 22, 2021 at 5:27
  • 1
    $\begingroup$ You can edit your question, and add/delete there any links you like. I suggest to add links to your previous relevant questions (like the link to the OEIS that is already given). $\endgroup$ Oct 22, 2021 at 9:34

1 Answer 1

2
$\begingroup$

The same idea of grouping terms by the number of unit bits, as well as grouping by the value of $\mathrm{wt}(j)$ (representing $j$ via individual bits) works here: \begin{split} s(n,m) &= \sum_{\ell=0}^n \sum_{t_1 + \dots + t_\ell \leq n-\ell} \sum_{j_1,\dots,j_\ell\in\{0,1\}} m^{\ell-\sum_{i=1}^\ell j_i} \prod_{k=1}^{\ell} (1+\sum_{i=1}^k j_i)^{t_k+1} \\ &=[x^{n+1}]\ \sum_{\ell=0}^n \sum_{j_1,\dots,j_\ell\in\{0,1\}} m^{\ell-\sum_{i=1}^\ell j_i} \prod_{k=0}^{\ell} \frac{(1+\sum_{i=1}^k j_i)x}{1-(1+\sum_{i=1}^k j_i)x} \\ &=[x^{n+1}]\ \sum_{\ell=0}^n \sum_{J=0}^\ell \sum_{j_1,\dots,j_\ell\in\{0,1\}\atop j_1+\dots+j_\ell=J} m^{\ell-J} \prod_{k=0}^{\ell} \frac{(1+\sum_{i=1}^k j_i)x}{1-(1+\sum_{i=1}^k j_i)x} \\ &=[x^{n+1}]\ \sum_{\ell=0}^n \sum_{J=0}^\ell m^{\ell-J} \sum_{d_0,d_1,\dots,d_J\geq 1\atop d_0+d_1+\dots+d_J=\ell+1}\prod_{k=1}^{J+1} \left(\frac{kx}{1-kx}\right)^{d_k} \\ &=[x^{n+1}]\ \sum_{\ell=0}^n m^{\ell+1}[y^{\ell+1}]\ \sum_{J=0}^\ell \prod_{k=1}^{J+1} \frac{kxy}{(1-kx-kxy)m} \\ &=[x^{n+1}]\ \sum_{J=0}^n \prod_{k=1}^{J+1} \frac{kx}{1-kx-kmx} \\ &=\sum_{J=0}^n (J+1)!\ [x^{n-J}]\ \prod_{k=1}^{J+1} \frac{1}{1-k(m+1)x} \\ &=\sum_{J=0}^n (J+1)!\ (m+1)^{n-J} \left\{n+1\atop J+1\right\}. \end{split} It also shows that the summands in the last formula corresponds to fixed values of $\mathrm{wt}(j)=J$.

$\endgroup$

Your Answer

By clicking “Post Your Answer”, you agree to our terms of service and acknowledge that you have read and understand our privacy policy and code of conduct.

Not the answer you're looking for? Browse other questions tagged or ask your own question.